LSAT and Law School Admissions Forum

Get expert LSAT preparation and law school admissions advice from PowerScore Test Preparation.

User avatar
 Dave Killoran
PowerScore Staff
  • PowerScore Staff
  • Posts: 5853
  • Joined: Mar 25, 2011
|
#63875
Complete Question Explanation
(The complete setup for this game can be found here: lsat/viewtopic.php?t=15078)

The correct answer choice is (C)

Please note that this question asks for the list of days on which Anastasia could park in the $10 lot in a single solution to the game, not the complete list of all possible days on which Anastasia could park in the $10 lot. Thus, the correct answer choice must be part of a single workable solution.

From our initial diagram we know that neither Wednesday nor Thursday can be a day on which Anastasia parks in the $10 lot. This information eliminates answer choices (D) and (E) from contention.

The remaining three answer choices involve Monday or Tuesday or both, and as discussed in the setup, there are no rules regarding Monday or Tuesday. Thus, at first it may appear that any of the three remaining answer choices could be the correct answer. At this point, if you were running out of time you should guess (C) because it is “different” than the other two answers in that it contains two days whereas the other two answers only contain one day. Answer choice (C) does turn out to be correct, but obviously it is better to select that answer based on facts as opposed to an educated guess. With that in mind, let’s use answer choice (A) to show why both answer choices (A) and (B) are incorrect.

Answer choice (A): If Monday is the only day on which Anastasia parks in the $10 lot, then from the numerical distribution, the $10 lot cannot be Z and it must be X or Y (lot Z must appear more than once in the five days). But, of course, from the second rule we know that lot X costs more than lot Z, and thus the $10 lot cannot be X, and therefore the lot that Anastasia parks in on Monday must be lot Y.

We can also infer that Friday will be a $12 lot (remember, there can be only one $10 lot, and Friday must be $10 or $12), and that therefore Wednesday will be a $15 lot (because the third rule stipulates that Wednesday’s lot costs more than Friday’s lot).
Further, because we have inferred that lot Y is the $10 lot, we know that Thursday’s $15 lot, which could only have been lot X or Y, must in fact be lot X. This deduction leads to the further deduction that lot Z is the $12 lot. This establishes the following relationship of lots to cost:

..... ..... ..... Lot X = $15
..... ..... ..... Lot Y = $10
..... ..... ..... Lot Z = $12

These deductions lead to the following setup:

O04_Game_#4_#22_diagram 1.png
However, astute test takers may have already noticed a problem—lot X has appeared twice in our setup and we have already established that X can only appear once in this game. Thus, assigning Monday as the only $10 lot does not allow for a viable solution to the game.

Answer choice (B): The reasoning used to eliminate answer choice (A) also applies to (B); simply shift the $10 lot to Tuesday and proceed with the same logic.

Answer choice (C): This is the correct answer choice.

Answer choice (D): Because Wednesday cannot be a $10 lot, this answer choice can be eliminated.

Answer choice (E): Because Thursday must be a $15 lot, this answer choice can be eliminated.
You do not have the required permissions to view the files attached to this post.
 salsaden
  • Posts: 8
  • Joined: May 02, 2012
|
#4270
In this game, I was unsure of one thing with question 22 which asks for a complete and accurate list of the days on which Anastasia parks in the $10 lot. I got the right answer (c)- but was wondering bc in question 20 I produced a hypothetical that had y (which I ascertained was the 10$ lot) to be the one Anastasia parked in on friday.
Just wondering if I did that hypothetical wrong or if the question stem is mistaken in saying this is a complete and accurate list ( i know that it is a could be true question- so is it referring to a possible scenario in which she parks in the $10 lot on both monday and tuesday?)
 Adam Tyson
PowerScore Staff
  • PowerScore Staff
  • Posts: 5153
  • Joined: Apr 14, 2011
|
#4279
Good question, salsaden, and you've already got the answer. Because question 22 is a could be true question, it's not asking for a complete and accurate list of every lot that could be the $10 lot. Rather, it's asking which of the answer choices could be all the $10 days. In other words, which of them is POSSIBLE, as opposed to which one is REQUIRED. Answers A and B can't be right, because the $10 lot can't happen just one of those days. If it did, the two numerical distributions (3-1-1 and 2-2-1) would prove that either Z doesn't happen more often than X or else X is not more expensive than Z.

D is easily eliminated using your not-law that says the $10 lot cannot be on Wednesday, since it has to be more expensive than the Friday lot, and E is out because we were already given that Thursday is a $15 day. Only C is possible.

Your other hypothetical is probably fine - I too have one with a $10 lot on Fridays, and it's Y in my hypo too. Since this is only about finding ONE possible answer, where the wrong answers are all impossible, it doesn't conflict with that other hypo.

Hope that helped! Good work!

Adam M. Tyson
PowerScore LSAT Instructor

Get the most out of your LSAT Prep Plus subscription.

Analyze and track your performance with our Testing and Analytics Package.